You are on page 1of 29

Krok 2 Stomatology 2008 1

1. A 3 year old child has a cavity in A. Pin stump inlay and a crown
the 84 tooth. Objectively: there is a cari- B. Pin tooth with a ring
ous cavity on the masticatory surface of C. Pin tooth with an inlay
the tooth within mantle dentin. Dentin is D. Pin tooth with protective bar
softened, probing of cavity walls is painful, E. Simplified pin tooth
percussion is painless. Make a diagnosis:
5. A 20 year old homeless and
A. Acute median caries unemployed patient complains about
B. Chronic superficial caries body temperature rise up to 39oC, pain
C. Acute superficial caries during eating and deglutition, nasal
D. Acute deep caries haemorrhages. He has been suffering
E. Chronic deep caries from this for 10 days. Objectively: herpetic
rash on the lips, irregular-shaped erosi-
2. A 45 year old patient complains ons covered with fibrinous deposit on
about pain induced by thermal stimuli, the mucous membrane of oral cavi-
spontaneous pain in the 26 tooth. A week ty; filmy deposits on the tonsils. Liver
ago this tooth was treated on account is enlarged and sclerotic. Blood count:
of pulpitis. Objectively: the 26 tooth is erythrocytes - 4, 5 · 1012 /l; hemoglobin
filled, percussion is painful, thermal sti- - 120 g/l; ESR - 25 mm/h; leukocytes -
muli induce long-lasting, slowly intensifyi- 10 · 109 /l; eosinophils - 0; rod nuclear
ng pain. X-ray picture shows that palatine cells - 2; segmentonuclear leukocytes - 31,
canal is filled by 2/3, no material can be lymphocytes - 41; monocytes - 10; atypical
seen in the buccal canals. What is the most mononuclears - 14%, plasmatic cells - 2.
probable cause of this complication? What is the most probable diagnosis?
A. Incomplete pulp extirpation A. Infectious mononucleosis
B. Inadequate canal filling B. Acute herpetic stomatitis
C. Infection C. Oropharyngal diphtheria
D. Inflammation in periodontium D. AIDS
E. Injury of periodontium tissue E. Acute leukosis
3. A 49 year old patient complains about 6. A 22 year old patient complained about
pain in the oral cavity induced by eating. colour change of the 11 tooth crown. The
He suffers from CHD. Objective exami- tooth was treated on account of chronic
nation revealed dyspnea, limb edema. deep caries a year ago. Immediately after
Oral cavity isn’t sanitated. On the mucous treatment the patient felt slight pain. She
membrane, on the right, there is an ulcer didn’t consult a dentist. X-ray picture
with irregular edges covered with greyish- shows broadening of periodontal fissure
white necrotic deposit with low-grade in the area of root apex of the 11 tooth.
inflammation around it. There is also Percussion is painless. What is the most
halitosis. What is the most probable di- probable diagnosis?
agnosis?
A. Chronic fibrous periodontitis
A. Trophic ulcer B. Chronic deep caries
B. Traumatic ulcer C. Chronic granulating periodontitis
C. Tuberculous ulcer D. Chronic granulematous periodontitis
D. Cancerous ulcer E. Chronic fibrous pulpitis
E. Ulcero-necrotic stomatitis
7. A 43 year old patient applied to a doctor
4. A 29 year old patient complains that the complaining about painful deglutition, li-
crown of his 26 tooth broke off. Objecti- mited mouth opening. 3 days ago he felt
vely: tooth decay index is 0,9, root ti- pain in the decayed 37 tooth, on the next
ssues are hard, roentgenography revealed day there appeared limitations of mouth
that the canal was filled to the top, no opening. Objectively: a slight soft swelli-
chronic inflammations of periodontium ng in the left submaxillary area. Palpation
were revealed. What construction should revealed an enlarged lymph node. Mouth
be used for prosthetics of the 26 tooth? can be opened up to 1,5 cm. There is
also an edema of the left palatine arch
and lateral pharynx wall. The 37 tooth is
decayed, mucous membrane around the
tooth is edematic, percussion is slightly
painful. What is the most probable di-
Krok 2 Stomatology 2008 2

agnosis? 11. A 42 year old patient complains of


pain in the submaxillary and sublingual
A. Odontogenous peripharyngeal areas that is getting worse during eating,
phlegmon body temperature rise up to 37, 6oC. He
B. Odontogenous pterygomaxillary has been suffering from this for 2 months.
phlegmon Objectively: infiltration along the right
C. Odontogenous submaxillary phlegmon sublingual torus, hyperemia, soft tissue
D. Peritonsillar abscess edema, acute pain during palpation. The
E. Acute submaxillary lymphadenitis duct of the right submandubular salivary
gland excretes turbid saliva mixed with
8. A 45 year old patient applied to pus. What is the most probable diagnosis?
a dental clinic for oral cavity sanati-
on. Anamnesis data: mild case of di- A. Exacerbation of salivolithiasis
abetes. Objectively: mucous membrane of B. Acute purulent lymphadenitis of
cheeks is unchanged, mainly in retromolar submaxillary area
area there are symmetrically placed C. Adenophlegmon of submaxillary area
whitish papulae protruding over the D. Abscess of maxillolingual groove
mucous membrane and forming a lace- E. Retention cyst of sublingual salivary
like pattern. On the upper jaw there are gland
two soldered bridge dentures, the 47 tooth
has an amalgam filling, the 46 tooth has a 12. A 38 year old patient came to a hospi-
steel crown. What is the most probable di- tal with complaints about a wound in the
agnosis? area of her left cheek. She was injured
16-18 hours ago, didn’t lose consciousness.
A. Lichen ruber planus Objectively: injury of skin, subcutaneous
B. Secondary syphilis fat and a muscle for about 3 cm long. The
C. Leukoplakia wound is bleeding. What initial surgical d-
D. Chronic atrophic candidosis bridement should be performed?
E. Lupus erythematosus
A. Early initial surgical d-bridement
9. A 38 year old patient was admitted B. Delayed surgical d-bridement
to the orthopaedic department after a C. Secondary surgical d-bridement
therapeutic treatment on account of peri- D. Initial surgical d-bridement together
odontitis. His anamnesis contains data with plasty
about frequent recurrences of this disease, E. -
acute attacks are accompanied by peri-
odontal abscesses. Objectively: the 42, 13. A 26 year old patient complains about
41,31, 32 teeth manifest mobility of I-II a sense of tooth heaviness and pain caused
degree, the 43 and 33 are stable, tooth by hot food stimuli, halitosis. Objecti-
cervices are dehisced by 1/4, hyperesthesia vely: crown of the 46 tooth is grey, there
of hard tooth tissues is not observed. What is a deep carious cavity communicating
rational construction of permanent splint with tooth cavity, superficial probing is
should be recommended? painless, deep one is painful, percussi-
on is painful, mucous membrane has no
A. Fullcast circle removable splint pathological changes. Make a provisional
B. Halfcrown splint diagnosis:
C. Mamlock’s splint
D. Crown-shell splint A. Chronic gangrenous pulpitis
E. Girder Kurliandky’s splint B. Chronic fibrous pulpitis
C. Acute condition of chronic periodontitis
10. A 52 year old patient complains D. Chronic concrementous pulpitis
of gum inflammation, tooth mobility. E. Chronic granulating periodontitis
Objectively: dentitions are intact, all teeth
have the 1-2 degree of mobility. What type 14. A 67 year old patient complains about
of teeth stabilization should be applied in frequently recurring erosion in the area
this case? of vermilion border of his lower lip. The
erosion is oval, 0,8х1,3 cm large, covered
A. Arch-directed stabilization with thin crusts. After their removal some
B. Frontal petechial haemorrhages on glossy surface
C. Parasagittal can be seen. There are also some atrophi-
D. Sagittal ed areas of the vermilion border. Infiltrati-
E. Frontosagittal on is absent. Submandibular lymph nodes
are not enlarged. What is your presumpti-
Krok 2 Stomatology 2008 3

ve diagnosis? vely: mucous membrane of lower lip is


hyperemic, excretory ducts of minor sali-
A. Abrasive precancerous Manganotti’s vary glands are dilated, "dew"symptom
cheilitis is present. What is the most probable di-
B. Erosive-ulcerous form of leukoplakia agnosis?
C. Keratoacanthosis
D. Bowen’s disease A. Cheilitis glandularis
E. Glandular cheilitis B. Meteorological cheilitis
C. Microbal cheilitis
15. A 51 year old woman complained D. Cheilitis exfoliativa
about food sticking in a tooth on the ri- E. Contact allergic cheilitis
ght of her lower jaw. Objectively: there
is a deep carious cavity on the distal- 19. A 54 year old woman complains about
masticatory surface of the 45 tooth consi- wear of the 35 and 36 teeth and pain
sting of compact pigmented dentin and caused by thermal and chemical stimuli.
not communicating with tooth cavity. The Objectively: crowns of the 35 and 36 teeth
patient was diagnosed with chronic deep are worn by 1/3 of their height (horizontal
caries. What diagnostic method allowed type), the 24, 25 26 teeth have fullcast
to eliminate chronic periodontitis? crowns. What crowns should be made for
the 35 and 36 teeth?
A. Electro-odontometry
B. Probing A. Fullcast
C. Palpation of root apex projection B. Combined Belkin’s
D. Percussion C. Metal stamped
E. Cold probe D. Plastic
E. Metal-ceramic
16. A 58 year old patient complains about
clicking in the mandibulotemporal joint 20. A virtually healthy 9 year old chi-
during eating. Periodically as a result of ld complains about crown fracture and
wide mouth opening the mandible takes pain in the right superior frontal tooth.
up such a position that makes mouth closi- Objectively: crown part of the 11 tooth is
ng impossible. The mandible can be easily broken by 1/3, pulp is pointwise dehisced,
placed into its initial position by pressing it is red, acutely painful and bleeds duri-
with fingers upon its coronoid processes. ng probing. Percussion is slightly painful.
What is your presumptive diagnosis? The child got a trauma several hours ago.
Choose an optimal treatment method:
A. Habitual dislocation of mandible
B. Acute dislocation of mandible A. Biological method
C. Subluxation of mandible B. Vital amputation
D. Anterior dislocation C. Devital amputation
E. Posterior dislocation D. Vital extirpation
E. Devital extirpation
17. An 11 year old girl complains about gi-
ngival haemorrhage during tooth brushi- 21. Objective examination of a patient
ng and eating. She has been sufferi- revealed hard tissue defect of the 24 tooth.
ng from this for a year. Gum of both Tooth decay index is 0,8. The defect is
upper and lower jaws is edematic and partly restored by a filling that doesn’t
congestively hyperemic. Hygienic state of meet the requirements. The tooth has
oral cavity is unsatisfactory. Bite is edge- pink colouring. X-ray picture shows no
to-edge. Roentgenological examination pathological processes. What constructi-
of periodontium revealed no pathologi- on is indicated in this case?
cal changes. What is the provisional di-
agnosis? A. Pin construction
B. Artificial crown
A. Chronic catarrhal gingivitis C. Inlay
B. Acute catarrhal gingivitis D. Halfcrown
C. Generalized periodontitis E. Equator crown
D. Localized periodontitis
E. Hypertrophic gingivitis 22. A 49 year old patient applied to
a dentist complaining about a growing
18. A 13 year old child complained about mole, its peeling and itching. According to
painfulness of his lower lip. He has the patient, the part of skin started changi-
been suffering for two months. Objecti- ng its colour and size 1 year ago, after an
Krok 2 Stomatology 2008 4

injury got during shaving. Objectively: in broken jaw with cartilage of nose
the infraorbital area on the left there is an and eyeballs, nasal haemorrhage, ti-
intensely pigmented brown spot, up to 2 ssue edema, "glasses"symptom; palpati-
cm large, with small nodules on its surface; on reveals crepitation, subcutaneous
it is oval, bulging, with signs of peeling, emphysema, liquorrhea, loss of sensitivi-
palpatory painless. Regional lymph nodes ty in the area of half the upper jaw, nose
are enlarged, adhering to skin, painless. wing and frontal teeth. What is the most
Make a provisional diagnosis: probable diagnosis?
A. Melanoma A. Le Fort’s III fracture of upper jaw
B. Pigmented nevus B. Upper jaw ostemia
C. Verrucous nevus C. Upper jaw odontoma
D. Squamous cell carcinoma of skin D. Le Fort’s I fracture of upper jaw
E. Papillomatosis E. Upper jaw cancer on the right
23. A patient applied to a dentist complai- 26. An 8 year old child has a carious cavi-
ning about inability to close her mouth, ty on the masticatory surface of the 16
laboured speech. Objectively: oral cavity tooth within circumpulpar dentin. Probi-
is half-open with saliva outpouring from ng of cavity floor is painful, dentin is
it, central line is deviated to the right. softened, slightly pigmented. Cold sti-
There is a cavity in front of antilobium; mulus causes short-term pain. Choose a
below the malar arch there is a protrusion dental treatment paste:
of mandible head into the infratemporal
fossa. What is the most probable di- A. Hydroxycalcium-containing
agnosis? B. Resorcin-formaline
C. Paraformaldehyde
A. Unilateral anterior dislocation of D. Arsenious
mandible on the left E. Thymol
B. Unilateral anterior dislocation of
mandible on the right 27. A 24 year old patient applied to a
C. Unilateral posterior dislocation of dental clinic for root removal of her upper
mandible on the left wisdom tooth. Tuberal anesthesia caused
D. Unilateral posterior dislocation of a postinjection haematoma. What was
mandible on the right injured during anesthetization?
E. Bilateral anterior dislocation
A. Pterygoid venous plexus
24. A 36 year old man complains about B. Maxillary artery
acute headache, body temperature rise up C. Infraorbital artery
to 39, 1o C, indisposition. Objectively: a sli- D. Zygomatic artery
ght face asymmetry because of soft tissue E. Palatine artery
edema of the left infraorbital area. Crown
of the 26 tooth is partly decayed. Percussi- 28. A 5 year old child has temperature rise
on is acutely painful. Mucous membrane up to 39, 2oC, sore throat, nausea. Objecti-
on the vestibular side in the area of vely: mucous membrane of soft palate and
the 25, 26 teeth is edematic, hyperemic. palatine arches is brightly hyperemic, it
Breathing through the left part of nose is can be distinctly distinguished among the
laboured, there are purulent discharges. surrounding tissues. The tongue is dry,
X-ray picture showed a homogeneous edematic, bright-red, its lateral surfaces
shadow of the left part of maxillary sinus. have no fur on them, fungiform papi-
What is the most probable diagnosis? llae are evidently enlarged. Face skin
is hyperemic apart of pale nasolabi-
A. Acute purulent odontogenous maxillary al trigone, is covered with spotty rash.
sinusitis Submaxillary lymph nodes are palpatory
B. Acute condition of chronic periodontitis painful. What is the causative agent of this
of the 26 tooth disease?
C. Acute periostitis of upper jaw
D. Suppuration of maxillary cyst A. Hemolytic streptococcus
E. Acute odontogenous osteomyelitis B. Coxsackie virus
C. Herpes virus
25. A 38 year old patient got a blow D. Bordet-Gengou bacillus
that resulted in upper jaw fracture. E. Loeffler’s bacillus
Objectively: flattening and impressi-
on of face, mobility and dangling of 29. A 65 year old patient complains about
Krok 2 Stomatology 2008 5

pain during eating, reduced tongue mobi- 32. A 31 year old man complains about
lity, an ulcer in the posterior part of mouth dryness, burning of tongue dorsum that
floor on the left, weight loss. Objecti- appeared for about a week ago and is
vely: the patient uses a partial removable getting worse during eating stimulati-
denture for the lower jaw. In the area ng food. Some time ago the patient
of the left mylohyoid duct there is an had pneumonia. He spent two weeks
ulcer in form of a cleft up to 1,6 cm at a hospital, was taking antibiotics. He
long with everted edges, covered with doesn’t take any drugs at the moment.
grey-yellowish deposit, closely adheri- Objectively: mucous membrane of oral
ng to an infiltrate that can be detected cavity is hyperemic, dry and glossy. On the
during bimanual palpation. In the left tongue dorsum and palate some greyish-
submandibular and superolateral areas of white films are present that can be easi-
neck several enlarged nonmobile lymph ly removed. Threads of saliva follow the
nodes can be palpated. What is the most spatula. What is the most probable provi-
probable diagnosis? sional diagnosis?
A. Cancer of mucous membrane of mouth A. Acute pseudomembranous candidosis
floor B. Chronic hyperplastic candidosis
B. Tuberculous ulcer of mouth floor C. Acute atrophic candidosis
C. Decubital ulcer of mouth floor D. Drug-induced stomatitis
D. Tertiary syphilis (gummatous ulcer) E. Chronic atrophic candidosis
E. Actinomycosis of mouth floor
33. A 10 year old boy applied to a dentist
30. A 7 month old child was brought to and complained about pain in his palate
a dentist because of an ulcer in the oral during eating. Objectively: the lower third
cavity. The child was born prematurely. of his face is shortened, mouth opening is
She has been fed with breast milk substi- not reduced. During joining of teeth the
tutes by means of a bottle with rubber ni- cutting edge of inferior incisors contacts
pple. Objectively: on the border between with mucous membrane of palate. On
hard and soft palate there is an oval ulcer the site of contact mucous membrane is
0,8х1,0 cm large covered with yellowish- hyperemic and slightly edematic. Lateral
grey deposit and surrounded with a roll- teeth have 1 class joining (according to
like infiltration. Make a provisional di- Angle’s classification). Propose a rational
agnosis: plan of treatment of the patient’s lower
jaw:
A. Bednar’s aphtha
B. Setton’s aphtha A. To "knock in"the frontal part
C. Tuberculous ulcer B. To "knock in"the lateral parts
D. Acute herpetic stomatitis C. To broaden lower jaw
E. Acute candidous stomatitis D. To lengthen the frontal part
E. To lengthen the lateral parts
31. A 16 year old patient complains
of a cosmetic defect in the area of his 34. A 32 year old patient applied to a
upper frontal teeth in form of white spots dental surgeon for oral cavity sanitation
that were revealed long ago and haven’t before prosthetics. During examination
changed since that. Objectively: there are of oral cavity the dentist revealed that
white spots on the vestibular surfaces of crown of the 35 tooth was decayed. The
the 11, 12, 21, 22 teeth by the cutting root is stable, its percussion is painless.
edge and on the vestibular surfaces of the Mucous membrane of alveolar process
16, 26, 36, 46 teeth close by the masti- was unchanged. X-ray picture showed a
catory surface. Probing showed that the slight broadening of periodontal fissure.
spot surface was smooth, painless; reacti- What is your presumptive diagnosis?
on to the cold stimulus was painless. The
spots couldn’t be stained by 2% soluti- A. Chronic fibrous periodontitis of the 25
on of methylene blue. What is the most tooth
probable diagnosis? B. Chronic periodontitis of the 25 tooth
C. Chronic granulomatous periodontitis of
A. Systemic enamel hypoplasia the 25 tooth
B. Local enamel hypoplasia D. Chronic granulating periodontitis of the
C. Acute initial caries 25 tooth
D. Fluorosis, spotty form E. Cystogranuloma
E. Erosion of hard tooth tissues
Krok 2 Stomatology 2008 6

35. A patient has indication for removal of A. Acute arsenous periodontitis


his medial incisor of the right upper jaw B. Acute infectious periodontitis
on account of chronic periodontitis. What C. Pulpitis complicated by focal peri-
types of anaesthesia should be applied for odontitis
tooth removal? D. Acute condition of chronic periodontitis
E. Acute purulent pulpitis
A. Infraborbital and incisive
B. Infraorbital and palatine 39. An 8 year old boy was referred to the
C. Terminal and incisive oral surgery for extraction of his 64 tooth
D. Infraorbital, palatine and incisive because of acute condition of chronic peri-
E. Plexual and incisive odontitis. Tooth crown is intact. What
instrument should be applied?
36. A 45 year old man complains about li-
quid outpouring from his nose, inability A. S-shaped forceps without thorns
to blow his nose, inflated cheeks. Objecti- B. Beak-shaped forceps with non-
vely: there is a perforating defect (1х1,5 converging beaks
cm) of alveolar process at a level of the C. Straight forceps
extracted 26th tooth in the lateral part D. Broad-beaked forceps
of his upper jaw. Air inhalation through E. S-shaped forceps with thorns
the nose with held nostrils is accompani- 40. A 43 year old man applied to the
ed by generation of bubbles in the area dental clinic for tooth prosthetics. Objecti-
of perforation. What denture constructi-
vely: the crown of the 37th tooth is
on should be recommended? decayed by 2/3, buccal and lingual walls
A. Lesser saddle denture with clasps’ are thin. Occlusion picture shows strong
fixation contact with antagonists. How thick
B. Clasp denture with obturating part should be the layer of tooth surface that
C. Common partial removable denture must be ground off during preparation for
D. Common dental bridge metallic stamped crown?
E. Protective palatal bars A. 0,28-0,3 mm
37. Preventive examination of an 8 year B. 0,1-0,2 mm
old boy revealed matted chalky spots on C. 0,5-0,6 mm
the vestibular surface of the 11 and 21 D. 0,6-0,7 mm
teeth localized in the precervical area. The E. 0,7-0,8 mm
child has no subjective complaints. What
41. A 56 year old man complains of
is the most probable diagnosis? pain in the left parotidomasticatory area,
A. Acute initial caries progressing face asymmetry that was noti-
B. Spotty fluorosis ced a month ago. Objectively: left-sided
C. Local enamel hypoplasia paresis of mimic muscles. To the fore of
D. Acute superficial caries earflap there is an ill-defined infiltration,
E. Chronic initial caries the skin above it is tense and cyanotic;
left lymph nodes are enlarged. Opening of
38. A patient complains about pain in mouth is limited down to 2,5 cm. The left
the 51 tooth that is getting worse duri- parotid duct doesn’t excrete saliva. What
ng cutting. Anamnesis data: the patient is the most probable diagnosis?
underwent treatment on account of pulpi-
tis of the 51 tooth, the tooth was treated A. Adenocarcinoma
with devitalizing paste, the patient didn’t B. Mixed tumour
come to see a dentist for the second time. C. Cyst of the gland
Objectively: carious cavity of the 51 tooth D. Glandular tuberculosis
is closed by dentin layer. Percussion is pai- E. Chronic lymphadenitis
nful. Mucous membrane in the root apex 42. A 55 year old woman complains of
projection of the 51 tooth is hyperemic, ulcers on the mucous membrane of oral
edematic, palpatory painful. Make a di-
cavity, pain during eating and talking. She
agnosis: fell abruptly ill over a month ago. Objecti-
vely: unchanged mucous membrane of
her gums, soft palate and palatine arches
has big bright-red erosions on it. Intact
mucous membrane peels easily off when
slightly rubbed and this results in erosi-
Krok 2 Stomatology 2008 7

ons and small haemorrhages. What is the A. Circumpulpar pin inlay


leading diagnostic symptom in the di- B. Metal-ceramic crown
fferential disease diagnostics? C. Halfcrown
D. Ceramic crown
A. Tzanck cells in the impression smears E. Metal crown
B. Positive Nikolsky’s symptom
C. Presence of blisters in the oral cavity 46. A teenager applied to an orthodonti-
D. Presence of Wickham’s striae st complaining about tooth malposition.
E. Presence of phagocytes in the impressi- Objectively: the face is without peculari-
on smears ties. Occlusion of permanent teeth is
present. There are no abnormalities of jaw
43. A 47 year old patient complains of correlation in three planes. The 23 tooth is
permanent pain in the 27 tooth that is vestibularly over the occlusive plane; the
getting worse during cutting. Objectively: space in the dental arch is less than 1/3
the patient’s face is symmetric, skin is of of crown size. How is it possible to make
normal colouring, mouth opening is not room for the malpositioned 23 tooth?
limited, mucous membrane of alveolar
process is edematic and hyperemic at a A. To remove the 24 tooth
level with the 27 tooth. The 27 tooth has B. To enlarge transversal jaw dimensions
a deep carious cavity communicating with C. To enlarge sagittal jaw dimensions
pulp chamber. Percussion of the 27 tooth D. To remove the 23 tooth
causes acute pain. What is presumptive E. To enlarge vertical dimensions
diagnosis?
47. A patient complains about acute
A. Acute condition of chronic periodonti- intense pain and a slight swelling in the
tis of the 27 tooth area of a decayed tooth of the lower jaw
B. Chronic periodontitis of the 27 tooth on the right, ill health, body temperature
C. Acute general purulent pulpitis of the rise up to 38, 3oC, lack of appetite,
27 tooth insomnia. Objectively: there is collateral
D. Acute purulent periostitis of the upper edema of soft tissues of submandibular
jaw beginning from the 27 tooth area and lower part of right cheek. Regi-
E. Chronic left-sided odontogenous maxi- onal lymph nodes are enlarged on the
llary sinusitis right, palpatory painful. Crown of the
46 tooth is half decayed, the 45, 46, 47
44. A 68 year old patient has a defect in teeth are mobile, there is also a cuff infi-
the lower third of his nose as a result of ltrate in the area of these teeth. Vincent’s
malignant tumour removal. Objectively: symptom is positive on the right. Make a
dorsum of nose up to the middle third diagnosis:
and external nostril outlines are intact.
Skin at the base of nose is thinned and A. Acute odontogenous osteomyelitis
erosive. The patient strongly objects to B. Acute purulent odontogenuos periosti-
any plastic operation. What method of fi- tis
xation of nose ectoprosthesis is the most C. Acute purulent periodontitis
acceptable? D. Acute condition of chronic
odontogenous osteomyelitis
A. On the glasses frame E. Chronic odontogenous osteomyelitis
B. By means of lace
C. Transparent adhesive tape 48. A 16 year old patient complained
D. Anatomic retention about discomfort in the area of her upper
E. Head cap jaw teeth she has been feeling for 2 weeks.
Examination of precervical area of the 11
45. A 12 year old girl complained about and 12 teeth revealed whitish matt spots
a crown defect in the frontal part of her with indistinct outlines that absorb dyes
upper jaw. Anamnesis data: the tooth intensively. What treatment of the 11 and
was filled more than once but the filli- 12 teeth should be administered?
ngs fell out. Objectively: the 12 tooth is
filled. There is IV class defect accordi- A. Remineralizing therapy
ng to Black’s classification. Devitalization B. Preparation and filling
has never been performed, percussion of C. Spot removal
the 12 tooth is painless. What orthopaedic D. Antiseptic treatment
construction should be applied in this E. Silver impregnation
case?
49. A 48 year old patient complained
Krok 2 Stomatology 2008 8

about having pain in the 45 tooth duri- Objectively: a painful elastic infiltration in
ng cutting for a year. The 45 was treated the left submaxillary area. Mouth openi-
before. Objectively: mucous membrane ng is not limited. Bimanual palpation in
in the area of this tooth is hyperemic the area of mylohyoid groove revealed
and slightly cyanotic. The 45 tooth is pi- a compact movable oblong induration.
nk, the filling fell out. What examinati- Mucous membrane is unchanged. Duct
on method should be applied in order to of the left submandibular gland doesn’t
choose treatment? excrete saliva. What is the most probable
diagnosis?
A. Roentgenography
B. Thermometry A. Salivolithiasis
C. Gum palpation B. Chronic lymphadenitis
D. Probing C. Pleomorphic adenoma
E. Electric odontodiagnostics D. Retention cyst
E. Submaxillary lipoma
50. A 40 year old patient with mandi-
ble fracture applied to a doctor 3 weeks 53. A 5 year old child was diagnosed wi-
after immobilization of breaks because of th congenital complete nonclosure of soft
pain and body temperature rise. Objecti- and hard palate. What type of anaesthesia
vely: a slight swelling in the chin area, is indicated for uranostaphyloplasty?
mucous membrane of alveolar process in
the area of the 2 1 | 1 2 teeth is hyperemic, A. Nasotracheal narcosis
edematic, palpatory painful. Overtooth B. Mask narcosis
splint on 5 4 3 2 1 | 1 2 3 4 5 teeth is C. Intravenous narcosis
in satisfactory condition, no occlusion D. Orotracheal narcosis
abnormalities were detected. The patient E. Endotracheal narcosis through
was diagnosed with acute purulent peri- tracheostome
ostitis of mandible. What surgical action is 54. A 23 year old woman came to a dental
indicated? clinic for restoration of the 11th tooth’s
A. Lancing of abscess to the bone crown. Objectively: root of the 11th tooth
B. Intraoral novocaine block is at a level with gingival edge, its walls
C. Removal and replacement of the are thick enough. A dentist made and
overtooth splint by a new one adjusted a stump inlay upon which a
D. Trepanation of the 2 1 and 1 2 teeth metal-plastic crown will be fixed. What
E. Supervision of patient plastic will be used for veneering?

51. A 43 year old patient complains of a A. Sinma-M


neoplasm in the right submaxillary area B. Acryloxide
that appeared a month ago after angi- C. Carbodent
na. Body temperature is 37, 0 − 37, 2oC. D. Noracryl
The patient underwent anti-inflammatory E. Protacryl
therapy but the neoplasm didn’t dimi- 55. A 35 year old patient needs a metal
nish. Objectively: palpation of the right inlay for the 37 tooth. Objectively: masti-
submaxillary area reveals a slightly pai- catory surface of the 37 tooth has a carious
nful spherical neoplasm of dense elastic cavity. What is the pecularity of preparati-
consistency with regular outlines that on?
is not adherent to skin. The duct of
submandibular salivary gland excretes A. Making a bevel
transparent saliva. Sublingual torus is B. Making an auxiliary shelf
unchanged. What is the most probable di- C. Broadening of cavity floor
agnosis? D. Making a flat floor
E. Making an auxiliary cavity
A. Chronic lymphadenitis
B. Chronic sialoadenitis 56. It is planned to make a metal-ceramic
C. Salivolithiasis crown supported by stump inlay of the
D. Salivary adenoma 23 tooth. Objectively: the crown of the
E. Atheroma 23 tooth is decayed down to the gingival
edge. Root canal is filled to the top. The
52. A 37 year old man complained about dentist made a wax stump model with a
pain and a slight swelling emerging duri- pin, cast it in metal, fitted it to the tooth,
ng eating in the left submaxillary area. fixed it by means of visphat-cement and
Krok 2 Stomatology 2008 9

got a working plaster impression. At what coalveolar crest, gaseous crepitation in the
stage did he make an error? right infraorbital area; percussion reveals
a "bursted nut"symptom. What is the
A. Impression taking most probable diagnosis?
B. Wax construction making
C. Casting A. Fracture of the right zygomatic bone
D. Stump fitting B. Le Fort’s I fracture of upper jaw (inferi-
E. Stump fixing or)
C. Le Fort’s II fracture of upper jaw
57. A 23 year old military servant needs D. Le Fort’s III fracture of upper jaw
orthopaedic treatment in a specialized (superior)
hospital. He was diagnosed with false joint E. Fracture of nose bones
of mandible in its frontal part. The teeth
are intact, stable, in threes on each side. 60. A patient is 48 year old, according
Orthopaedic treatment by means of a bri- to the results of clinicoroentgenological
dge denture will be possible only if the jaw examination it is indicated to remove
defect is no more than: the 26 tooth because of acute condition
of chronic granulomatous periodontitis.
A. 1 cm What conduction anesthesia is indicated
B. 2 cm for this operation?
C. 3 cm
D. 3,5 cm A. Tuberal and palatinal
E. 4 cm B. Torus
C. Infraorbital and incisive
58. A 17 year old girl applied to a dental D. Plexus
clinic and complained about hard tissue E. Infraorbital and palatinal
defects on her frontal and lateral teeth.
Subjectively these defects don’t cause any 61. A girl is 1,2 year old. Vestibular surface
inconvenience. Crown defects appeared of her 52, 51, 61, 62 teeth has large carious
long ago. The patient was born and and cavities within the enamel. Probing is sli-
has been living in an area where fluori- ghtly painful, percussion of the 52, 51, 61,
ne concentration in the drinking water 62 teeth is painless. What treatment is to
makes up 1,2 mg/l. Objectively: on the be administered?
vestibular surfaces of incisors on both
upper and lower jaws in the equator area A. Silver impregnation
there are hard tissue defects within deep B. Filling with phosphoric acid cement
layers of enamel. The defects are parallel C. Remineralizing therapy
to the cutting edge. The same defects were D. Coating with fluorine lacquer
revealed in the area of tubera of the fi- E. Amalgam filling
rst molars, floor and walls of the defects
are smooth. Enamel of the defect floor is 62. A 2 year old girl has body temperature
light-brown. What is the most probable 38, 5oC, a swelling below her jaw on the
diagnosis? right. On the 5th day of illness there
apeared rhinitis, cough, a small movable
A. Systemic hypoplasia globule under her lower jaw on the ri-
B. Local hypoplasia ght. Objectively: general condition of the
C. Focal odontodysplasia child is moderately severe. The face is
D. Endemic fluorosis asymmetric due to the swelling in the right
E. Erosion of hard tissues of tooth submaxillary area. The skin is hyperemic,
glossy, there is a diffuse infiltrate in the
59. A 19 year old patient came to right submaxillary area spreading to the
traumatology centre and complai- upper neck parts on the right, it is dense
ned about face asymmetry, right-sided and painful; the skin doesn’t make folds.
paresthesia of his upper lip, nasal Teeth are healthy. What is the most
haemorrhage. Objectively: evident face probable diagnosis?
asymmetry due to an edema and
haematoma of the right inferior eyelid
and infraorbital area. Opening of mouth
is slightly limited, occlusion is normal.
Palpation reveals a symptom of "step"in
the area of the right zygomaticomaxillary
suture, deformity in the area of external
edge of the right orbit and zygomati-
Krok 2 Stomatology 2008 10

A. Adenophlegmon of the right submaxi- roots is intact, there are no roentgenologi-


llary area cal changes in this area. What is the most
B. Acute nonodontogenic submandibular probable diagnosis?
lymphadenitis on the right
C. Acute purulent periostitis of mandible A. Acute diffuse pulpitis
on the right B. Chronic fibrous pulpitis
D. Acute sialoadenitis of the right C. Chronic gangrenous pulpitis
submandibular salivary gland D. Exacerbation of chronic periodontitis
E. Chronic osteomyelitis of mandible on E. Acute deep caries
the right
66. A 12 year old boy complains about
63. A 36 year old woman complains of lip pain and swelling in the parotidomasti-
dryness and peeling lasting for a month. catory area on the left, body temperature
Application of indifferent ointments was rise up to 37, 5oC. He has been suffering
ineffective. Objectively: vermilion border from this for 5 years. Objectively: palpati-
of her lower lip is bright-red, moderately on reveals a dense painful nonmobi-
infiltrated, covered with adherent whitish- le formation 3,5х5 large in the paroti-
grey squamae, their removal causes pain domasticatory area on the left. Skin
and haemorrhage. On the focus periphery colour is unchanged. Orifice of the left
there is epithelium opacity in form of whi- salivary gland duct excretes transparent
te striae, in the centre of it there is a sinki- secretion. What is the most probable di-
ng down area. What is the most probable agnosis?
diagnosis?
A. Herzenberg’s pseudoparotitis
A. Lupus erythematosus B. Mixed tumour of parotid gland
B. Candidal cheilitis C. Epidemic parotitis
C. Lichen ruber planus D. Acute condition of chronic
D. Leukoplakia parenchymatous parotitis
E. Exfoliative cheilitis E. Buccal abscess
64. A patient complained about pai- 67. A 55 year old patient has a painless,
nful deglutition, difficult mouth openi- tuberous, cyanotic pedunculated formati-
ng. Several days ago the 47 tooth was on 2х1х1,5 cm large that appeared on the
removed because of acute condition of spot of the removed 46th tooth. Opening
chronic periodontitis. The patient’s condi- of mouth is not limited. Intra-oral X-ray
tion kept worsening. Body temperature is picture of alveolar process in the area of
37, 9oC. Results of external examination: the removed 46th tooth shows a focus of
the face is symmetric, face skin is slightly bone tissue destruction. What is the most
pale. Right submandibular lymph nodes probable diagnosis?
are enlarged, palpatory painful. Exami-
nation of oral cavity is impossible because A. Giant-cell epulis
of evident contracture of lower jaw (the B. Hard odontoma of lower jaw
mouth opens up to 0,5 cm between central C. Hypertrophic gingivitis
incisors). What anesthesia will provide D. Papilloma of mucous membrane in the
mouth opening? area of the removed 46th tooth
E. Ameloblastoma of the lower jaw
A. Bersche-Dubov’s anesthesia
B. Plexus 68. A 40 year old woman complains
C. Torus about slight painfulness of gums, high
D. Block of upper cervical plexus temperature sensitivity of teeth. Objecti-
E. Mandibular anesthesia vely: the gums are pale and compact,
in the area of lower frontal teeth there
65. A 7 year old child complains of is retraction with cervix dehiscence. X-
spontaneous pain in the upper right molar ray picture shows steady decrease of
teeth. Examination of medial contact interdental septa height down to 1/3
and masticatory surfaces of the 55 tooth of root length. What disease are these
revealed a carious cavity composed of symptoms typical for?
softened light dentin and localized wi-
thin circumpulpar dentin. Floor probing
is acutely painful, tooth percussion is sli-
ghtly painful. Mucous membrane of the
alveolar process in projection of 55 tooth
Krok 2 Stomatology 2008 11

A. Parodontosis is palpatory unchanged. What is the most


B. Generalized periodontitis probable diagnosis?
C. Gingivitis
D. Localized periodontitis A. Arthrosis of the right temporomandi-
E. Papillitis bular joint
B. Acute arthritis of the right
69. A soldier injured by a shell splinter temporomandibular joint
was diagnosed with gunshot fracture of C. Neuromuscular syndrome
mandible accompanied by an over 3 cm D. Occlusive articulation syndrome
long bone defect in the chin area. What E. Subluxation of mandible
method of fixation of mandible fragments
is indicated? 72. During opening a phlegmon of mouth
floor a doctor revealed greyish necrotic
A. Machine osteosynthesis (Rudko, masses in purulent foci, gas vesicles and
Bernadsky) fat droplets, sharp unpleasant smell of
B. Tigerstedt’s splints exudate. The tissues are of dark-brown
C. Direct osteosynthesis colour, muscles resemble of boiled meat.
D. Gunning-Port’s splint What medications should be administered
E. Intermandibular Ivy ligature in order to prevent further spreading of
this process?
70. A 43 year old patient complained
about mobility of his 24, 26, 27 teeth, A. Polyvalent antigangrenous serum
pus excretion from alveolus of the the B. Challenging dose of broad spectrum
extracted 25 tooth. 1,5 month ago di- antibiotics
ssection along the mucogingival fold C. Glucocorticoid medications
was performed and the 25 tooth was D. Hyposensitizing medications
extracted. Objectively: there is a sli- E. Immunomodulators
ght swelling of soft tissues in the right
infraorbital area, lymph nodes of the ri- 73. A 43 year old patient complai-
ght submaxillary area are enlarged, sli- ned about mobility of lower jaw teeth.
ghtly painful, nasal breathing is normal. Objectively: the dentition is intact. Tooth
Mucous membrane of alveolar process in mobility is of I-II degree. It is planned to
the area of the 24, 26, 27 teeth is edematic immobilize teeth by means of a removable
and cyanotic. There is also a fistula with splint common for the whole dentition.
bulging granulations along the mucogi- What stabilization will be provided by
ngival fold. Alveolus of the extracted means of this splint?
25 tooth excretes purulent granulations.
What disease does this clinical presentati- A. Circle
on correspond with? B. Frontal
C. Frontal-lateral
A. Chronic localized osteomyelitis D. Sagittal
B. Acute osteomyelitis E. Transversal
C. Acute condition of localized periodonti-
tis 74. A patient complains about long-lasting
D. Acute condition of chronic maxillary pain attacks in the lower jaw teeth, on the
sinusitis left. The pain irradiates to the ear, occiput
E. Chronic alveolitis and is getting worse during eating cold
and hot food. Objectively: there is a deep
71. A 57 year old patient came to an carious cavity on the approximal-medial
orthopaedic stomatology center with surface of the 36 tooth. Floor probing is
complaints about dull pain in the area of overall painful and induces a pain attack.
his right temporomandibular joint that What is the most probable diagnosis?
is getting worse during eating. The di-
sease developed gradually, it began wi- A. Acute diffuse pulpitis
th constrained mandibular motion in B. Acute local pulpitis
the morning that sometimes grew more C. Acute purulent pulpitis
intensive, sometimes less. Objectively: D. Chronic concrementous pulpitis
the face is symmetric, the mouth can be E. Acute deep caries
opened up to 3 cm. Opening of mouth is 75. A 35 year old woman complains about
accompanied by articular noise and cli- periodical appearance of small ulcers in
cking (step-like dislocation of mandible). the oral cavity. She has been suffering
Skin above the joint is intact. Muscle tone from this for 5 years, recurrences happen
Krok 2 Stomatology 2008 12

4-5 times a year. The ulcer healing lasts months. Objectively: on the masticatory
for 10 days. Objectively: on a mucous surface of the 37 tooth there is a carious
membrane of lower lip there is a roundi- cavity with overhanging enamel edges fi-
sh lesion element 0,5 cm large covered lled with circumpulpar dentin. The cavity
with white deposit and surrounded by is filled with light softened dentin. Probi-
hyperemia border, very painful when ng of the cavity floor is somewhat painful.
touched. What is the most probable di- Cold stimuli cause short-term pain. Make
agnosis? a provisional diagnosis:
A. Chronic recurrent aphthous stomatitis A. Acute deep caries
B. Traumatic erosion B. Chronic fibrous pulpitis
C. Secondary syphilis C. Acute median caries
D. Chronic recurrent herpes D. Chronic median caries
E. Duhring’s herpetiform dermatitis E. Chronic deep caries
76. A 38 year old driver complains 79. A 65 year old patient complains of
of acute pain in his mouth and sore pain in the area of mucous membrane of
throat, difficult ingestion, indisposition, hard palate on the left that is getting worse
temperature rise up to 38, 6o C. These during eating with use of a complete
symptoms appeared after exposure to removable denture. He has been suffering
cold. He has been ill for a day, in the from this for 1,5 month. Objectively: left-
evening he has to go to work. Objectively: sided hyperemia and edema of mucous
gums in the area of inferior frontal teeth membrane of hard palate; at the border
as well as pharynx mucous membrane of distal denture edge there is an ulcer
are hyperemic, edematic; gingival edge with dense walls and fundus, surroundi-
is necrotic, tonsills are enlarged. Results ng tissues are infiltrated. The ulcer floor is
of bacterioscopy: fusospirochetal symbi- tuberous, covered with fibrinous deposit;
osis. In blood: erythrocytes - 4, 5 · 1012 /l; ulcer palpation is painful. What exami-
leukocytes - 7, 2 · 109 /l; ESR - 18 mm/h. nation method is to be applied in the first
What actions should a stomatologist take? place?

A. To prescribe a medication and give a A. Biopsy


sick-list B. Bacterioscopy
B. To prescribe a medication and allow to C. Cytology
go to work D. Allergic contact plastic test
C. To give him an order for hospitalization E. Serological reactions
in oral department
D. To refer him to an infectious disease 80. A 46 year old patient complains about
specialist pain and bleeding from the carious cavi-
E. To refer him to an otolaryngologist ty of her 27 tooth during eating. Previ-
ously she had spontaneous pain. Exami-
77. A 25 year old man complains about nation of the 27 tooth revealed a deep
itching and reddening of his skin in the carious cavity on the masticatory surface
buccal area, general weakness, flaccidi- consisting of red tissue, probing induced
ty. He associates the begin of disease pain and haemorrhage. What treatment
with a skin injury he had got during method should be chosen?
shaving. Objectively: body temperature is
39, 0oC. In the buccal area a part of skin is A. Vital extirpation
hyperemic, slightly bulging, well-defined. B. Devital extirpation
Hyperemic surface has some vesicles wi- C. Devital amputation
th serous fluid. What is the most probable D. Vital amputation
diagnosis? E. Biological method

A. Erysipelatous inflammation of face 81. A 40 year old patient complains about


B. Buccal phlegmon a carious cavity in the 22 tooth. Objecti-
C. Furuncle vely: a deep carious cavity on the medial
D. Anthrax surface of the 22 tooth, probing induces
E. Streptococcal impetigo mild pain. What is the optimal material
for filling of the 22 tooth?
78. A 13 year old child has been sufferi-
ng from pain in the left inferior molar
induced by cold stimuli for several
Krok 2 Stomatology 2008 13

A. Composite light-setting material is 2 degree tooth mobility. Dentogingival


B. Silica-alumina cement pockets are 4-5 mm deep. External exami-
C. Glass-ionomer light-setting cement nation revealed dryness and thickening
D. Glass-ionomer chemical-setting cement of palms, anterior third of forearms, sole
E. Silicophosphate cement surfaces; there are scratches. What is the
most probable provisional diagnosis?
82. A 28 year old man applied to a dental
surgeon for removal of the 38 tooth. What A. Papillon-Lefevre syndrome
forceps should be chosen for this tooth? B. Gaucher’s disease
C. Letterer-Siewe disease
A. Beak-shaped curved forceps D. Hand-Schueller-Christian disease
B. Broad-beaked forceps (with non- E. Niemann-Pick disease
converging beaks)
C. Beak-shaped forceps with converging 86. Examination of a patient revealed
beaks a roundish neoplasm in the submental
D. Beak-shaped forceps with thorns area. The skin above it is unchanged,
E. Root bayonets forms a fold. Puncture sample contai-
ns some straw-yellow liquid mixed with
83. A 35 year old patient applied to a cholesterol. What is the most probable di-
dental clinic for removal of the 14 tooth agnosis?
because of acute condition of chronic
periodontitis after therapeutic treatment A. Congenital median cyst
proved to be inefficient. What instrument B. Lipoma
will you choose for removal? C. Lymphadenitis
D. Retention cyst of sublingual salivary
A. S-shaped forceps gland
B. Bayonet-shaped crown forceps E. Dermoid cyst
C. S-shaped right forceps
D. Straight forceps 87. An 8 year old child complains about
E. Bayonet-shaped root forceps permanent dull pain in the 46 tooth that
is getting stronger during cutting. The
84. A patient came to a dental clinic for pain appeared 1 day ago. Previously there
the purpose of prosthetics. Objectively: has been pain induced by cold stimuli.
the lower third of her face is diminished, Objectively: there is a deep carious cavi-
nasolabial folds are deepened, frontal ty on the masticatory surface of the 46
group of teeth on both upper and lower tooth, tooth cavity is closed, probing and
jaws is missing, crowns of the 17, 15, 26, 27, temperature stimuli cause no pain reacti-
36, 37, 45, 47 teeth are worn by 2/3 of their on. Percussion is painful, a slight mobility
height, masticatory surfaces of these teeth is present. Gum around the 46 tooth is
are smooth, pigmented, alveolar process is hyperemic, edematic, palpatory painful.
not hypertrophied, interalveolar height is X-ray picture shows no changes near the
reduced. What form of pathological tooth apex of undeveloped roots. What is your
wear is it? provisional diagnosis?
A. Horizontal, noncompensated, III A. Acute serous periodontitis
degree of severity B. Acute purulent pulpitis
B. Horizontal, compensated, II degree of C. Acute general serous pulpitis
severity D. Acute purulent periodontitis
C. Vertical, noncompensated, III degree of E. Acute condition of chronic periodontitis
severity
D. Vertical, compensated, III degree of 88. A 20 year old patient complains of
severity a tumour-like formation on the lateral
E. Mixed, noncompensated, III degree of surface of his neck on the right. He
severity revealed this tumour a year ago. Objecti-
vely: there is a semi-oval well-defined
85. An 11 year old patient complai- tumour 3х2 cm large on the lateral neck
ns about tooth mobility and gingival surface. The tumour doesn’t hold together
haemorrhage. He has been suffering si- with skin, it is painless, dense and elastic,
nce he was 3 years old. Objectively: upwardly and sidewardly movable. Regi-
gums around all the teeth are hyperemic, onal lymph nodes are not enlarged. After
edematic, bleed during instrumentl the tumour puncture some light yellow
examination. Tooth roots are dehisced by liquid was obtained that consisted of
1/3 and covered with white deposit. There cast-off epithelium, cholesterol crystals,
Krok 2 Stomatology 2008 14

lymphocytes, erythrocytes. What is the the 42, 41, 31, 32 teeth are dehisced by
most probable diagnosis? 2/3, there is pathological mobility of the
III degree. The patient has indication for
A. Lateral neck cyst removal of the 42, 41, 31, 32 teeth. What
B. Median neck cyst type of denture should be applied for
C. Dermoid neck cyst immediate-insertion prosthetics?
D. Neck chemodectoma
E. Chronic neck lymphadenitis A. Partial removable lamellar
B. Clasp
89. A 28 year old woman complains about C. Adhesive
acute pain on the left of her upper jaw D. Ceramic-metal bridge
during eating. 6 days ago the 28 tooth E. Stamped-soldered bridge
was extracted, after that the pain came.
The patient considers that extraction of 93. A 13,5 year old girl complains of gingi-
the 28 tooth caused "damaging"of the val painfullness and haemorrhage duri-
next tooth. Objectively: alveolus of the ng tooth brushing and eating, halitosis.
28 tooth is at a stage of healing. On the She has been ill with angina for a week.
distal surface of the 27 tooth there is a Objectively: mucous membrane of gums
carious cavity in the precervical area that in the area of frontal teeth of her upper
doesn’t communicate with the tooth cavi- and lower jaws is edematic, hyperemic.
ty. Cold stimulus causes short attack of Apices of gingival papillae are necrotic,
pain. Dentin of walls and floor is light and they also bleed when touched. There is a
softened. Probing of the floor is painful. thick layer of soft tooth plaque. What is
What is the most probable diagnosis? the causative agent of this disease?
A. Acute deep caries A. Anaerobic microflora
B. Acute local pulpitis B. Herpes virus
C. Acute diffuse pulpitis C. Streptococci
D. Chronic deep caries D. Staphylococci
E. Chronic fibrous pulpitis E. Yeast fungi
90. A 68 year old patient underwent full 94. A month after cementation of a metal-
jaw removal. Before the operation an ceramic crown on the 23 tooth a pati-
impression of the patient’s upper and ent applied to the dentist with complai-
lower jaws was taken and a substitutive nts about its decementation. Examination
denture was made. What are the means of revealed that tooth stump was of sufficient
the denture fixation in the oral cavity? height, its walls converged to the vertical
tooth axis at an angle of approximately 30
A. Spiral Fosher springs degrees. At what angle was it necessary to
B. Anchors establish convergention of stump walls to
C. Clasps the tooth axis?
D. Magnets
E. Attachments A. Up to 8 degrees
B. 12-15 degrees
91. A patient complained about pain in his C. 15-18 degrees
45 tooth induced by cold, sour and sweet D. 22-25 degrees
food stimuli. The pain abates when the E. 10-12 degrees
stimulus action is stopped. Objectively:
there is a carious cavity on the masticatory 95. A 57 year old patient complains about
surface within mantle dentin consisting of mobility of his metal-ceramic dental bri-
food rests and softened dentin, overhangi- dge supported by the 33, 37 teeth. The bri-
ng enamel edeges are chalky. What is the dge has been in use for 9 months. Objecti-
diagnosis? vely: X-ray picture shows alveolar process
atrophy by 2/3 in the area of the 33, and
A. Acute median caries by 1/2 of root length in the area of the 37;
B. Chronic median caries there are pathological pockets, gingivitis.
C. Acute superficial caries What is the cause of pathological mobility
D. Acute deep caries of supporting teeth?
E. Chronic deep caries
92. A 48 year old teacher complained
about considerable mobility of the 42,
41, 31, 32 teeth. Objectively: cervices of
Krok 2 Stomatology 2008 15

A. Functional shifting of supporting teeth A. Physiological infantile retrogenia


under stress B. Mesial occlusion
B. Injuring of circle ligament by crown C. Physiological occlusion
edges D. Distal occlusion
C. Devitalization of supporting teeth E. Edge-to-edge occlusion
D. Massive grinding off of hard tissues
E. Garland modelling in the precervical 100. A 52 year old patient complains
area of significant tooth wear on both jaws.
Objectively: tooth wear in the lateral
96. During lateral motions of a mandi- parts of the lower jaw at a level of gums
ble frontal teeth are deviated. Sideward and wear by 1/3 in the frontal parts. The
relocation of incisive point from the treatment had two stages. What denture
central position has an angle of 100 −110o . construction for occlusion disconnection
What is characterized by such value of and reconstruction of myostatic reflexes
incisive point deviation? should be applied in this case?
A. Lateral incisive tract A. Dentogingival splint in the lateral parts
B. Lateral articulate tract B. Plastic cap for the whole dentition
C. Sagittal incisive tract C. Plastic cap in the frontal part
D. Sagittal articulate tract D. Crown splint in the lateral parts
E. Bennett’s angle E. Elastic plastic cap
97. A child was brought to the 101. A 1,8 year old boy was under
traumatology centre of oral surgery treatment in the infectious disease
department with complaints about department. He was given ampicillin. On
changed position of the 21 tooth that was the 6th day of treatment there appeared
inclined towards palate. The day before white deposits in form of caseous films
the child was hit in the face. Make a di- that were revealed on the hyperemic
agnosis: mucous membrane in the area of gingi-
val torus, cheeks and on the tongue. The
A. Subluxation of the 21 tooth films can be removed leaving hyperemic
B. Complete dislocation of the 21 tooth surface underneath them. General condi-
C. Fracture of crown part of the 21 tooth tion is satisfactory. Body temperature is
D. Contusion of the 21 tooth 36, 7oC. What is the provisional diagnosis?
E. Break-off of the crown part of the 21
tooth A. Acute candidous stomatitis
B. Drug-induced stomatitis
98. Parents of a 1,5 year old child complai- C. Chronic candidous stomatitis
ned about tongue enlargement, disturbed D. Acute herpetic stomatitis
food intake. The child has ben sufferi- E. Mild case of leukoplakia
ng from this since birth. Objectively:
general condition has no pecularities. 102. A 47 year old patient came to
Tongue is enlarged (macroglossia). Its an orthopaedic stomatology center wi-
mucous membrane has granular vesicle- th complaints about missing tooth in the
like outgrowths. Tongue is compact, frontal part of his upper jaw, cosmetic
palpatory painless. What is the most defect. Objectively: the occlusion is
probable diagnosis? orthogenic, the 11th tooth is missi-
ng. Anamnesis data: the patient had
A. Lymphangioma of tongue myocardial infarction 3 months ago. What
B. Tongue hemangioma denture should be applied for the time
C. Tongue fibroma being?
D. Tongue cyst
E. Tongue cancer A. Partial removable lamellar denture
B. Bridge denture suported by the 21 and
99. A visiting nurse examined a newborn 12 teeth
child. Examination revealed that lower C. Bridge denture suported by the 21 tooth
face part is shorter than median one, chin D. Clasp denture
is retrodeviated, teeth are missing, lower E. Implant
jaw is retrodisplaced. What is the name of
such mandible position of a newborn? 103. A 53 year old patient has to undergo
resection of the left half of mandible
together with its ramus. It is indicated
to make an immediate Oxman’s denture.
Krok 2 Stomatology 2008 16

What denture element keeps the remai- Tooth crowns are intact but have oral posi-
ning mandible fragment from deviation tion. Complete joining of teeth is impossi-
towards the defect? ble because the teeth are situated beyond
the dental arch. X-ray picture shows a sli-
A. Removable or non-removable inclined ght broadening of periodontal fissure of
plane the 11 tooth up to 0,5-2 mm. The roots are
B. The whole fixing part intact. Make a correct diagnosis:
C. Multiclasp system
D. Resection part A. Traumatic subluxation of the 11, 21
E. Artificial teeth teeth
B. Traumatic periodontitis of the 11, 21
104. Examination of a 9 year old patient teeth
revealed a milky spot on the vestibular C. Traumatic complete dislocation of the
surface of the 11 tooth close to the cutti- 11, 21 teeth
ng edge. Probing and tempearture stimuli D. Fracture of alveolar process in the area
cause no pain reaction. The child was di- of the 11, 21 teeth
agnosed with local enamel hypoplasia of E. Traumatic extraction of the 11, 21 teeth
the 11 tooth. What treatment should be
administered? 108. A 50 year old patient has a defect of
his lower dentition. It is planned to make
A. A course of remineralizing therapy a bridge denture supported by implants.
B. Prophylactic hygiene of oral cavity X-ray picture showed that the height of
C. Spot removal osseous tissue mass from projection of
D. Hermetization of the affected part mandibular canal to the top of alveolar
E. Cosmetic filling crest was 2 cm. What implant wil be
recommended?
105. A 70 year old patient is awaiting
complete removable dentures for both A. Screw
upper and lower jaws. Teeth placement B. Endodonto-endoossal
will be made by Vasilyev’s method. What C. Leaflike
teeth in the upper denture must not touch D. Subperiosteal
glass? E. Conical
A. Lateral incisors and second molars 109. External examination of a 7 year old
B. Central incisors and first molars child revealed: thickening of nose bridge,
C. First and second premolars semi-open mouth, dry lips. Mouth corners
D. Canines and first molars are peeling. Anamnesis data: the child
E. Second premolars and first molars sleeps with open mouth. Examination of
oral cavity revealed no changes. What di-
106. Examination of a 6 year old child spensary group will this child fall into?
revealed a deep carious cavity in the 85
tooth. Percussion and probing are pai- A. The second
nless. Removal of softened dentin resulted B. The first
in intercommunication with tooth cavi- C. The third
ty. Deep probing is painful. X-ray picture D. The fourth
of the 85 tooth shows a focus of bony ti- E. -
ssue destruction in the area of bifurcati-
on, cortical plate of the 35 tooth has no 110. A 14 year old patient applied to
pathological changes. What paste should an orthodontist. Objective examination
be applied for the root filling in this case? revealed that on the site of the second
incisor a canine tooth had cut out, and on
A. Zink-eugenol paste the site of the canine - the second incisor.
B. Resorcin-formaline paste The same pathology has also the patient’s
C. Glass-ionomer cement father. Make a diagnosis:
D. Phosphate cement
E. Calcium-containing paste A. Transposition of lateral incisor and
canine
107. A 20 year old patient got a trauma in B. Distal position of lateral incisor
the area of his upper jaw. He applied to a C. Palatine position of lateral incisor
dentist and complained about mobility of D. Mesial position
his frontal upper teeth, pain during cutti- E. Superocclusion of incisor and
ng and joining of teeth. Objectively: the 11 infraocclusion of canine
and 21 teeth have II-III degree mobility.
Krok 2 Stomatology 2008 17

111. Parents of an 8 year old boy complain fficult food mastication, esthetic face
about a cosmetic defect, inability to bi- defect. Objectively: the face is proporti-
te off food. The child often suffers from onal, asymmetric as a result of oral
acute viral respiratory infections. Objecti- displacement of a lateral fragment of
vely: chin skewness, mental fold is most the left lower jaw. When the jaws are
evident. The lower lip is everted, superior closed the 34, 35, 36, 37 teeth are not in
central incisor lies on it, nasolabial fold contact with their antagonists. The pati-
is flattened. In the oral cavity: occlusion ent strongly objects to surgical operati-
period is early exfoliation period. The on. What orthopaedic treatment should
upper jaw is narrowed, there is gothic be administered?
palate. Frontal teeth have fan-shaped
position. Sagittal fissure is 6 mm. In the A. A denture with double dentition
lateral parts contact of homonymous teeth B. Mechanotherapy
is present. What is the most probable C. Interjaw traction
cause of dentoalveolar deformity? D. Crowns with occlusal applications
E. Levelling of occlusal surface
A. Pathology of upper airways
B. Missing of Caelinski ledge 115. A patient complains about
C. Endocrinal diseases spontaneous pain in the area of his 15
D. Untimely sanitation of oral cavity tooth he has been feeling for 2 days.
E. Gestational toxicosis Thermal stimuli make the pain worse, its
attacks last up to 30 minutes. Objecti-
112. A 20 year old patient complains vely: there is a deep carious cavity in
about gum itching, gingival haemorrage the 15 tooth consisting of light softened
during tooth brushing and eating, unusual dentin, floor probing is painful in one
look of gums. He has been observing these point, reaction to the thermal stimuli is
presentations for the last 1,5 year. It is positive, percussion is painless. Make a di-
known from the patient’s anamnesis that agnosis:
he has been taking diphenylamine anti-
convulsants for 2 years. Objective exami- A. Acute local pulpitis
nation revealed gingival hyperemia and B. Acute diffuse pulpitis
edema. In the frontal part the gums cover C. Pulp hyperemia
vestibular surface of teeth by 1/2 of their D. Acute deep caries
height. Along the loose gingival edge E. Acute condition of chronic pulpitis
growth of granulation tissue is present,
probing causes gingival haemorrage. No 116. A 50 year old woman complains
roentgenological changes were revealed. about a neoplasm on her lower lip on
Make a diagnosis: the side of oral cavity that appeared a
month ago and has been slowly growing
A. Chronic hypertrophic gingivitis since that. Objectively: there is a roundish,
B. Chronic ulcerative gingivitis elastic, painless neoplasm inside the lower
C. Chronic catarrhal gingivitis lip. Mucous membrane hasn’t changed its
D. Localized periodontitis colour. Make a diagnosis:
E. Generalized periodontitis
A. Retention cyst of lower lip
113. A patient has got a traumatic fracture B. Lip abscess
of mandible in the area of the missing 34, C. Lip papilloma
35 teeth with a slight displacement and a D. Lip fibroma
defect of alveolar part in the area of the E. Lip lipoma
34, 35 teeth. Other teeth on both lower
and upper jaws are intact. What splint 117. A 5 year old girl with crossbite was
would be optimal in this case? referred to an orthodontist. Objectively:
between frontal teeth there are diaereses
A. Tigerstedt’s splint with a spreading and diastems, canine tubera have no si-
curve gns of physiological wear out. Central line
B. Plain splint cramp between incisors doesn’t match. What is
C. Port’s splint the doctor’s tactics?
D. Vasilyev’s splint
E. Vankevich splint
114. A 36 year old patient applied to
a dentist 1,5 month after a mandi-
bular trauma with complaints about di-
Krok 2 Stomatology 2008 18

A. To remove unworn tubera of canines A. Fusospirochetal symbiosis


B. To administer jaw massage B. Streptostaphylococci
C. To make a screw plate for the upper jaw C. Pale treponema
D. To wait for autoregulation D. Virus of herpes simplex
E. To disconnect occlusion E. Candida fungi
118. Prophylactic examination of a 6 year 121. A 48 year old patient complained
old child revealed: occlusion of temporary about posttraumatic pain in the area of
teeth. Both superior and inferior dental his chin. Examination revealed median
arches are trapeziform. Superior incisors fracture of lower jaw with I class dentition
overlap the inferior ones by more than defect (according to Kennedy classificati-
2/3. Correlation of canines and second on). It is indicated to make a Vankevich
molars is homonymous. There are no splint in Stepanov’s modification. What
spaces between frontal teeth. Superior will determine the height of directing
dental arch is larger than inferior one by planes of this splint?
the size of buccal cusp. In what planes can
the occlusion deformity be defined? A. Extent of mouth opening
B. It will be determined arbitrary
A. Sagittal and vertical C. Height of central jaw correlation
B. Sagittal and transversal D. Form of edentulous alveolar crests of
C. Sagittal and occlusal the lower jaw
D. Sagittal and nasal E. Height of molar teeth of the upper jaw
E. Sagittal and Frankfort’s
122. A 24 year old soldier injured by a
119. A 43 year old patient complains of shell splinter was diagnosed with fracture
tooth mobility and considerable dehi- of mandible accompanied by an over 2 cm
scence of dental cervices of the lower jaw long bone defect in the chin area. What
in its frontal part. Objectively: gums in method of fixation of mandible fragments
the area of the 44, 43, 42, 41, 31, 32, 33, 34 is indicated?
teeth are pale, slightly cyanotic, without
haemorrhage. The 42, 41, 31, 32 teeth have A. Break fixation by means of Rudko’s
mobility of the I-II degree. There is also apparatus
overcrowding of the 42, 41, 31, 32 teeth. B. Tigerstedt’s splints
Cervices of the 42, 41, 31, 32 teeth are C. Intermandibular Ivy ligature
dehisced by 1/2 of root length, of the 43, D. Gunning-Port’s splint
33 - by 1/4. What orthopaedic constructi- E. Direct osteosynthesis
on should be applied in this case?
123. A 35 year old patient consulted a
A. Fullcast removable kappa dentist about a painless, slowly growi-
B. Kurlandsky’s girder splint ng neoplasm in the area of the 11 and
C. Cap splint 12 teeth. Examination revealed that
D. Halfcrown splint the neoplasm had flattened form, was
E. Semicircle splint adjacent to the teeth, had a pedicle, was
of light-pink colour, up to 1,5 cm large, wi-
120. An 18 year old patient complains th smooth surface and dense consistensy.
of gingival painfulness and haemorrhage, Diagnosis: epulis in the area of the 11, 12
halitosis, temperature rise up to 38, 6oC, teeth. What form of epulis are these clini-
general weakness, appetite loss. Objecti- cal presentations typical for?
vely: mucous membrane of oral cavity is
hyperemic and dry; tongue is covered with A. Fibrous
white fur, gingival papillae are edematic, B. Angiomatous
their apices have areas of necrotic deposit C. Giant-cell
that can be easily removed leaving bleedi- D. Epulis of pregnancy
ng surface beneath. Submaxillary lymph E. -
nodes are enlarged, palpatory painful.
What is the causative agent of this di- 124. As a result of a road accident a 45
sease? year old patient got an injury of his upper
jaw. Examination revealed elongated and
flattened face, profuse nasal haemorrhage,
liquorrhea from the nose and ears. These
clinical presentations are typical for the
following fracture of upper jaw:
Krok 2 Stomatology 2008 19

A. Subbasal (Le Fort III) A. Devital amputation


B. Subnasal (Le Fort I) B. Biological method
C. Suborbital (Le Fort II) C. Vital extirpation
D. Bilateral fracture of zygomatic bones D. Vital amputation
E. - E. Devital extirpation

125. A child is 2,5 year old. The parents 129. An 8 year old child complains about
complain about thumb sucking during pain in the 21 tooth that is getting worse
sleep. What tactics should the doctor during cutting. A month ago a part of
choose? tooth crown broke off as a result of a fall.
The child didn’t consult a dentist. Objecti-
A. To recommend an ulnar fixator vely: in the area of medial angle of the 21
B. To talk with a child about harm from tooth there is a crown defect that makes
thumb suction up 1/3 of the crown’s height. Tooth cavi-
C. Medical intervention is unnecessary ty is open, probing and thermal stimulus
D. Non-removable device for suppression cause no pain. Percussion is acutely pai-
of bad habit nful. Gum around the 21 tooth is edematic
E. Removable device for suppression of and hyperemic. What is the provisional di-
bad habit agnosis?
126. A patient is waiting for a partial A. Acute condition of chronic periodonti-
removable laminar denture. Anatomic tis
models of both jaws were made by means B. Acute condition of chronic pulpitis
of elastic alginate material "Ipin"and C. Acute serous periodontitis
referred for disinfection. What disinfecti- D. Acute purulent periodontitis
on method should be applied? E. Pulpitis complicated by periodontitis
A. Glutaraldehyde 2,5% рH 7,0-8,7 130. An orthodontist was treating a pati-
B. Sodium hypochloride 0,5% ent infected with AIDS virus. Acci-
C. Desoxone 0,1% dentally he injured skin of one of his
D. Hydrogen peroxide solution 6% fingers with a dental disk during tooth
E. Alcohol solution 70% preparation. What actions should the
orthodontist take in this case?
127. A 13 year old boy complains about
pain in the 46 tooth induced by cold sti- A. To press blood out and treat the skin
muli. Objectively: there is a deep carious with 70% alcohol solution
cavity on the masticatory surface of the 46 B. To apply a tourniquet on the shoulder
tooth within light softened circumpulpar C. To perform diathermo-coagulation
dentin. Probing of carious cavity floor is D. To treat the skin with 5% iodine soluti-
overall painful, cold stimulus causes acute on
pain that is quickly relieved after the sti- E. To press blood out and treat the skin
mulus’ elimination. Choose an optimal with strong solution of KMnO4
dental treatment paste:
131. A 36 year old patient complains
A. Hydroxycalcium-containing about acute pain, "clicking"in the right
B. Thymol mandibulotemporal joint, burning in the
C. Iodoform area of her right external acoustic meatus.
D. Resorcin-formaline Movements of her lower jaw are step-
E. Zink-eugenol like, along with brief blocking moments
in the joint and acute pain. Objecti-
128. An 8 year old child has a deep cari- vely: the face is symmetric. Occlusion is
ous cavity communicating with tooth cavi- orthognathic, intraoral palpation of lateral
ty on the distaloapproximal surface of the pterygoid muscle causes pain on the right.
75 tooth. Probing is painful, percussion is Tomograms show that contours of bone
painless, cold water causes slowly abating structures of articular surfaces are regular
pain. The tooth decayed a few months ago, and smooth. What is the most probable
wasn’t treated. What treatment method is diagnosis?
to be applied in this case?
Krok 2 Stomatology 2008 20

A. Mandibulotemporal joint disfunction


B. Rheumatic arthritis of mandi- A. With dentin wetting agent
bulotemporal joint B. Enamel and dentin should be treated
C. Acute posttraumatic artritis of mandi- with mordanting gel once again
bulotemporal joint C. Enamel and dentin should be treated
D. Deforming arthrosis of mandi- with adhesive system
bulotemporal joint D. Enamel and dentin should be treated
E. Anchylosis of mandibulotemporal joint with hydrogen peroxide
E. Enamel and dentin should be treated
132. A 40 year old patient complains with alcohol
about difficult mastication as a result of
lower jaw deviation. Anamnesis data: 136. A 42 year old patient complains
mental fracture 2 months ago. Objecti- about tooth mobility, change of tooth
vely: the 35; 36; 38 ... 45; 46 teeth are position, foul taste, pain during food
missing. Remaining teeth are intact. The mastication. Objectively: teeth of both
43; 44; 47; 48 teeth have no contact with upper and lower jaws are mobile (2-3
antagonists and their oral deviation makes degree), there are also diastems and di-
up 1 cm. What is the optimal construction aereses. Tooth roots are dehisced by 1/2.
of lower jaw denture? There are serous purulent discharges from
periodontal pockets. Orthopantomogram
A. A denture with double dentition shows bony tissue destruction down to 2/3
B. Metal-ceramic dental bridge of root length. There are bone pockets.
C. Soldered splint on rings What is the most probable diagnosis?
D. Removable lamellar denture
E. Adhesive denture A. Chronic generalized periodontitis (III
degree)
133. A patient is 24 years old. Artificial B. Papillon-Lefevre syndrome
crown of his 22 tooth broke off. The tooth C. Chronic catarrhal gingivitis
was devitalized 8 years ago and its crown D. Atrophic gingivitis
is totally decayed. What microprosthesis
should be made for restoration of tooth E. Chronic generalized periodontitis (II
crown? degree)

A. Stump-root inlay 137. A 60 year old man has got a pustule


B. Filling on his chin skin that quickly developed
C. Inlay into a hard, strongly painful infiltrate 3х3
D. Application cm large. In the epicentre three necrosis
E. Stamped crown zones around hair follicles are present.
Lymph nodes of chin are enlarged, pai-
134. One of the methods to define central nful. Body temperature is 38, 5oC. What is
occlusion is to construct a flat that will the most probable diagnosis?
go through cutting edges of central inci-
sors and distalobuccal tubercles of last A. Chin carbuncle
molar teeth providing that there is suffi- B. Erysipelatous inflammation of chin
cient quantity of teeth. What flat should C. Suppurated atheroma
be constructed? D. Chin furuncle
E. Allergic chin dermatitis
A. Occlusal flat
B. Sagittal 138. A 50 year old patient comlains of
C. Vertical difficult mastication, tooth mobility, hali-
D. Transversal tosis, gingival haemorrhage. Objectively:
E. Frankfort gums are hyperemic, cyanotic, there is also
dental scale. Parodontal pockets of upper
135. During restoration of a II class molar teeth were 8 mm, of the rest teeth -
(according to Black’s classification) cari- 6 mm. X-ray picture showed resorption of
ous cavity of the 25 tooth a doctor applied bone tissue by 2/3-1/2 of root length. What
methods of total mordanting and drying is the most probable diagnosis?
of hard tissues. After that he noticed that
enamel surface became chalky and dentin
became completely dry. How should he
treat enamel and dentin surfaces for the
further restoration by method of ”wet −
Bonding”?
Krok 2 Stomatology 2008 21

A. Generalized III degree periodontitis, A. Chronic initial caries


chronic B. Fluorosis
B. Generalized II degree periodontitis, C. Local enamel hypoplasia
chronic D. Acute initial caries
C. Generalized I degree periodontitis, E. Chronic superficial caries
chronic
D. Generalized III degree periodontitis, 142. A 35 year old man complains about
acute condition pain in the area of the 38 tooth, painful
E. Generalized II degree periodontitis, deglutition, difficult mouth opening. What
acute condition anesthesia method will be optimal during
operation on account of pericoronaritis?
139. An 18 year old patient complains
about gingival enlargement, haemorrhage A. Conduction Bersche-Dubov’s
and pain during eating. Objectively: gi- anesthesia
ngival edema, hyperemia, enlargement is B. Infiltration anesthesia
up to 1/3 of tooth crown length. Palpati- C. General anesthesia
on causes intense haemorhage and pain. D. Stem anesthesia
What is the most probable diagnosis? E. Application anesthesia
A. Hypertrophic gingivitis, edematous 143. A young woman complained about
form having vesicles in oral cavity, painful
B. Hypertrophic gingivitis, fibrous form ulcers, especially during talking and
C. Acute catarrhal gingivitis eating; muscle and joint pain, body
D. Chronic catarrhal gingivitis temperature rise, indisposition, weakness.
E. Acute condition of initial generalized She fell suddenly ill 2 days ago. Objecti-
periodontitis vely: to - 38, 4oC. Vermilion border is
covered with bloody crusts that stick
140. A boy is 1 month old. He has a together and impede mouth opening.
wound with purulent discharge near the Mucous membrane of lips, cheeks, mouth
medial edge of inferior eyelid, on the floor, tongue, soft palate is hyperemic
right. The illness began abruptly, body and edematic; there are single vesicles
temperature is up to 40o C. General condi- and large painful erosions covered with
tion is grave. On the second day of illness fibrinous deposit on it. Regional lymph
there appeared an infiltration near the nodes are enlarged and painful. There is
internal edge of orbit and cheek on the hypersalivation. Nikolsky’s symptom is
right. Skin above it is hyperemic, fluctuati- negative. What is the most probable di-
on cannot be detected. Palpebral fissure is agnosis?
narrowed. Right nasal meatus discharges
pus. There is an infiltration on the vesti- A. Multiform exudative erythema
bular surface of alveolar process and B. Acantholytic pemphigus
palate on the right. Mucous membrane C. Secondary syphilis
above it along the mucogingival fold is D. Acute herpetic stomatitis
hyperemic, there is fluctuation. What is E. Non-acantholytic pemphigus
the most probable diagnosis?
144. A 20 year old patient complains
A. Acute hematogenous osteomyelitis about gingival pain and haemorrhage
B. Acute dacryocystitis she has been suffering from for a week.
C. Right-side orbital phlegmon The pain is getting worse during eati-
D. Acute right-side maxillary sinusitis ng, haemorrhage is usually induced by
E. Acute serous periostitis mechanical stimuli. Objectively: there are
evident hyperemia and edema of gums
141. A 25 year old patient complains of in the area of inferior frontal teeth. Api-
a light brown spot on the frontal tooth ces of gingival papillae are dome-shaped,
of her upper jaw. Oblectively: a single enlarged. Gum palpation causes pain, sli-
light brown spot in the precervical area ght haemorrhage. No roentgenological
of the 23 tooth, probing revealed that its changes were revealed. What is the most
surface was smooth. Reaction to the cold probable diagnosis?
stimulus and probing was painless. What
is the most probable diagnosis?
Krok 2 Stomatology 2008 22

A. Catarrhal gingivitis A. 10-20% solutions of EDTA sodium


B. Hypertrophic gingivitis salts
C. Localized periodontitis B. Resorcin-formaline liquid
D. Ulcerative gingivitis C. 30% solution of silver nitrate
E. Atrophic gingivitis D. Essential oils
E. Medications with antibacterial and
145. A 42 year old patient applied to anti-inflammatory effect
a dentist and complained about whi-
te caseous deposit on the dorsum of 149. A patient applied to a dentist and
his tongue and burning sensation. It is complained about periodical pain in his
known from the patient’s anamnesis that upper jaw. Anamnesis data: the 12 tooth
he underwent treatment in an in-patient has been previously treated on account
hospital on account of pneumonia. What of pulpitis. Objectively: crown of the 12
is the most probable diagnosis? tooth was restored by permanent filling
material. X-ray picture of the upper jaw
A. Acute pseudomembranous candidosis shows a focus of bony tissue destruction
B. Typical form of leukoplakia near the root apex of the 12 tooth up to
C. Lichen ruber planus 15 mm in diameter. Root canal is filled
D. Scarlet fever up to the top. What is the most optimal
E. Lupus erythematosus treatment method?
146. A patient complained about A. Root apex resection of the 12 tooth
frequent haemorrhages from the mucous B. Conservative treatment
membrane of oral and nasal cavities, he C. Removal of the 12 tooth
mentioned also that his father had the D. Replantation of the 12 tooth
same problems. Objectively: there are E. Dissection along the mucogingival fold
multiple telangiectasias and angimatous
formations on face skin as well as on 150. A 46 year old patient complai-
mucous membrane of nose, cheeks and ns about difficult mouth opening,
lips. Blood count is normal. What is the body temperature rise, edematic tissues
most probable diagnosis? around both mandibulotemporal joints.
Anamnesis data: 2-3 months ago the pati-
A. Rendu-Osler-Weber disease ent had undurable mild bilateral pain
B. Werlhof’s disease attacks in the parotidomasticatory areas
C. Vaquez disease that lasted for a few days, limited mouth
D. Cushing’s basophilism opening, sense of tension and discomfort
E. Addison-Biermer disease in some areas of left and right mandi-
bulotemporal joint. What is the most
147. Unused stomatological instruments probable provisional diagnosis?
were left on a sterile table in the dental
room after the end of working day. What A. Rheumatic arthritis
actions should be taken in order to B. Infectious arthritis
provide sterility of these stomatological C. Arthrosis
instruments? D. Fibrous anchylosis
E. Deforming arthrosis
A. Sterilization without pretreatment
B. Disinfection, sterilization 151. An 11 year old girl has adentia,
C. Disinfection, presterilizing cleansing, the 35 tooth is missing, it was proved
sterilization roentgenologically. Between the 34 and
D. Presterilizing cleansing, sterilization 33 teeth as well as between the 34 and 36
E. Disinfection only teeth there are diaereses, the 34 tooth is
turned by 30o relative to its glossobuccal
148. A 23 year old patient underwent direction. What abnormal position does
treatment of pulpitis of her 16 tooth. Duri- the 34 tooth have?
ng endodontic procedure an obstructed
distal buccal canal was found. What medi- A. Tortoocclusion (rotation of teeth) and
cations can be applied for chemical di- distal
latation of root canal? B. Distal
C. Mesial
D. Vestibular
E. Oral
152. A 64 year old patient applied to a
Krok 2 Stomatology 2008 23

dental clinic for tooth prosthetics. Objecti- have surgery hours makes up 26 m2 . What
vely: there is a sharp bony prominence area must this room with two universal
in the area of the missing 15 tooth. It dental devices have according to the exi-
is planned to make a partial removable sting regulations?
denture with two-layer base. What plastic
should be used for elastic backing? A. 14 m2 for each dental device and 10 m2
additionally
A. Plastic PM-01 B. 10 m2 for each dental device and 10 m2
B. Ftorax additionally
C. Acryl C. 10 m2 for each dental device and 7 m2
D. Protacryl additionally
E. Bacryl D. 7 m2 for each dental device and 7 m2
153. A dentist was called to a patient bei- additionally
ng treated in the cardiological department E. 20 m2 for each dental device and 12 m2
after myocardium infarction. The pati- additionally
ent was diagnosed with acute conditi-
on of chronic fibrous pulpitis of the 36 157. Examination of a 23 year old
tooth. What method of pulpitis treatment patient revealed chronic candidosis of
should be chosen taking into account oral mucous membrane, generalized
grave condition of the patient? lymphadenopathy. Anamnesis data: the
patient has been suffering from herpes for
A. Devital amputation a year. Body temperature persistently ri-
B. Vital extirpation ses up to 37, 4 − 37, 5oC, body weight has
C. Vital amputation reduced by 8 kg over the last month. What
D. Devital extirpation disease can be indicated by this symptom
E. Conservative method group?

154. Walls and floor of a dental room were A. AIDS


painted with recommended light colours. B. Infectious mononucleosis
What is the required coefficient of reflecti- C. Acute leukosis
on from the walls, ceiling and floor? D. Chronic leukosis
E. Candidosis
A. 60
B. 35 158. A 51 year old lecturer came to the
C. 30 orthopaedic department and complained
D. 25 about painfullness and mobility of his
E. 20 frontal teeth of the lower jaw. Cervices
of the 42, 41, 31, 32 teeth are dehisced,
155. A 7 year old girl was brought to a III degree mobility with deep pathologi-
hospital for the purpose of oral cavity cal pockets is present. When would it be
sanitation. She was born and has been livi- reasonable to fix dentures after dental
ng in an area where fluorine concentration extraction?
in water makes up 2,5 mg/l. Examination
revealed symmetrically placed dark spots A. On the day of dental extraction
on the vestibular surfaces of the 11, 21, B. In 16-30 days
31, 41 teeth as well as on the tubera of the C. In 2-3 months
16, 26, 36, 46 teeth. It is known from the D. In 4-6 months
anamnesis that the teeth cut out already E. In 5-6 days
with affection. What is the most probable
diagnosis? 159. A 32 year old patient lost his 41
and 31 teeth as a result of a sporti-
A. Tooth fluorosis ng accident. Replantation is impossible
B. Odontogenesis imperfecta because these teeth have cracks and splits
C. Amelogenesis imperfecta both on crown and root surfaces. Clinical
D. Systemic enamel hypoplasia and roentgenological examination proved
E. Local enamel hypoplasia the possibility of immediate implantati-
on. What implant materials should be
156. A dentist applied to a regional preferred?
sanitary-and-epidemiologic institution for
an authorization to open a private dental
surgery with 2 universal dental devices.
The area of room where he is planning to
Krok 2 Stomatology 2008 24

A. Titanium membrane of lips and tongue roundi-


B. Carbon sh painful erosions 1-3 mm in diameter
C. Ceramic covered with whitish deposit. The gum on
D. Platinum both upper and lower jaws is hyperemic,
E. Sapphire edematic, bleed when touched. What is
the most probable diagnosis?
160. A 9 year old child complains about
pain and swelling of soft tissues beneath A. Acute herpetic stomatitis
his mandible on the right. Objectively: B. Stevens-Johnson syndrome
general condition is satisfactory; there is C. Recurrent aphthous stomatitis
face asymmetry due to the inflammatory D. Multiform exudative erythema
infiltration and collateral tissue edema E. Recurrent herpetic stomatitis
in the right submandibular area. Crown
of the 85 tooth is decayed, mucous 164. An 11 year old girl complains about
membrane of gums in the area of the bleeding of a tooth on her lower jaw
84, 85, 46 teeth is hyperemic, edematic; during eating and tooth brushing. This
mucogingival fold is flattened. What tooth hurt her before but she didn’t
treatment should be administered? consult a dentist. Examination of the 46
tooth revealed a deep cavity communi-
A. Extraction of the 85 tooth cating with tooth cavity and filled with
B. Endodontic treatment of the 85 tooth red growth tissue. Probing causes a slight
C. Extraction of the 85 tooth and incision haemorrhage and pain, percussion is pai-
along the mucogingival fold nless, cold stimulus cause mild pain. What
D. Periosteotomy is the most probable diagnosis?
E. Endodontic treatment of the 85 tooth
and periosteotomy A. Chronic hypertrophic pulpitis
B. Chronic granulating periodontitis
161. Mother of a 3 year old child brought C. Chronic papillitis
the child to an orthodontist and complai- D. Gingival polyp
ned about total lack of crown part of the E. Chronic simple pulpitis
51 and 61 teeth. What tactics should the
doctor choose? 165. A patient came to a dental clinic
for dental prosthetics. Objectively: total
A. Thin-walled cap lack of teeth on the mandible. Sharp and
B. Metal-ceramic crown regular atrophy of alveolar part. Frenula
C. Stump tooth attachment and fold position is high.
D. Inlay Name the type of atrophy of edentulous
E. Tooth extraction mandible using Keller’s classification:
162. A 12 year old girl complains about A. II type
burning and painfulness of her tongue, B. I type
especially during eating spicy food. C. III type
Objectively: there are oval red spots on D. IV type
the tip and dorsum of tongue. Filiform E. V type
papillae are not present in the affected
area. The girl mentions that the spots 166. A 35 year old patient complained
become periodically larger and have mi- about toothache on the left of his upper
gratory nature. What is the most probable jaw that appears during eating, can be
diagnosis? caused by thermal stimuli (especially by
cold water) as well as by mechanical and
A. Glossitis areata exfoliativa chemical stimuli. The pain abates when
B. Median rhomboid glossitis the stimuli are eliminated. Objectively:
C. Lingua plicata there is a deep carious cavity with a
D. Glossotrichia narrow inlet within circumpulpar dentin.
E. Raspberry tongue Probing of carious cavity floor is painful.
Thermodiagnosis causes acute pain that
163. A 2 year old child was brought
to a dentist to consult about rash and abates immediately after stimulus elimi-
nation. Electric odontodiagnostics results
painfulness during eating that appeared
- 15 microampere. Make a diagnosis:
yesterday. Objectively: body temperature
is 37, 5oC, skin is clean, submaxillary
lymph nodes are painful. Examination
of oral cavity revealed on the mucous
Krok 2 Stomatology 2008 25

A. Acute deep caries A. Mandibulofacial dysostosis


B. Pulp hyperemia B. Oculocerebrorenal syndrome
C. Acute partial pulpitis C. Oculoauriculovertebral dysplasia
D. Acute median caries D. Oculomandibulofacial syndrome
E. Chronic simple pulpitis E. Oculodentodigital syndrome
167. After anesthetization a 55 year old 171. A 23 year old man complai-
patient felt sudden attack of weakness, ns about gingival haemorrhage duri-
pain behind his breastbone irradiating to ng tooth brushing, massive formation
his left arm and scapular area, palpitati- of dental deposit in spite of thorough
on. Objectively: the patient is conscious, oral hygiene. Objectively: gingival papi-
inert, his forehead is covered with cold llae are somewhat edematic, congesti-
sweat, the skin is pale, AP is 90/60 mm vely hyperemic, bleed when touched.
Hg, heart tones are dull, pulse is thready Fyodorov-Volodkina hygienic index is 3,5.
and arrhythmic. What state is developing? What toothpaste would you recommend
this patient as a part of complex therapy
A. Myocardium infarction of this disease?
B. Cardiogenic form of anaphylactic shock
C. Stenocardia attack A. Toothpaste with salts
D. Collapse B. Toothpaste with mineralizing
E. - components
C. Fluorine-containing toothpaste
168. During tooth extraction a 55 year D. Gel toothpaste with microelements
old patient felt dull pain behind her E. Toothpaste with antifungal agents
breastbone, got a sense of compressi-
on. A dental surgeon diagnosed her wi- 172. A dentist is filling a II class carious
th a stenocardia attack. What medication cavity (Black’s classification) in the 36
should be given this patient in order to tooth of a 35 year old patient by sandwi-
arrest this state? ch method. What glass-ionomer cement
should be chosen as a basic liner in this
A. Nitroglycerine, validol case?
B. Analgin
C. Baralgin A. Hybrid
D. Dimedrol B. Water-setting
E. Ketanov C. Classic
D. Condensable
169. A 58 year old patient applied to an E. Hardened
oral surgeon and complained about pai-
nful ulcer on the lateral surface of his 173. A 19 year old man complained about
tongue. Objectively: left lateral surface of gingival haemorrhage and painfulness.
tongue has a roundish ulcer with undermi- Objectively: in the frontal part of his
ned soft overhanging edges, palpatory pai- upper and lower jaws overcrowding of
nful, ulcer floor is slightly bleeding and teeth is present; there are hyperemia,
covered with yellowish nodules. What is cyanosis and enlargement of gingival
the most probable diagnosis? papillae up to 1/3 of crown height; massi-
ve formation of soft tooth deposit. What
A. Tuberculosis procedure should be included into the
B. Syphilis treatment program?
C. Traumatic ulcer
D. Actinomycosis A. D’arsonval currents
E. Trophic ulcer B. Diathermy
C. Microwave therapy
170. An oral surgeon attended a 3 D. Ultra-high frequency therapy
day old child staying in the newborn E. Fluctuating currents
pathology department. Objectively: bi-
lateral hypogenesis of zygomatic bones 174. During local anesthetization (with
and orbits, antimongoloid slant of 2 ml of 10% solution of lidocaine) a 9
palpebral fissures, nonclosure of inferi- year old girl cried out, lost consciousness,
or eyelids, hypogenesis of mandible (bird there appeared generalized convulsions.
face), deformity of auricles with a preauri- Objectively: the child’s skin is pale and
cular fistula on the right. What congenital cyanotic. It is impossible to feel the pulse
disease does the child have? because of convulsions. What is your
provisional diagnosis?
Krok 2 Stomatology 2008 26

A. Acute hematogenous osteomyelitis


A. Lidocaine intoxication B. Phlegmon of infraorbital area
B. Quincke’s edema C. Acute maxillary sinusitis
C. Anaphylactic shock D. Acute odontogenous osteomyelitis
D. Bronchial obstruction E. Acute purulent periostitis
E. Febrile convulsions
179. A 62 year old man complains about
175. A 60 year old patient has profuse a painless formation on his tongue that
alveolus haemorrhage after a simple appeared several months ago. Objecti-
operation of the 37 tooth extraction. It is vely: there is a big number of carious
known from the patient’s anamnesis that and completely decayed teeth, a painless
he had been suffering from hypertension whitish formation 10х5 mm large with
for the latest 6 years. Arterial pressure irregular surface in form of verrucas on
is 180/110 mm Hg. What emergency aid the lateral surface of tongue. Histologi-
should the doctor render? cal examination revealed thickening of
corneous layer of epithelium with intermi-
A. To introduce hypotensive medications ttent cornification. What is the most
and perform tight alveolar packing probable diagnosis?
B. To perform alveolar packing with a
hemostatic sponge A. Verrucous form of leukoplakia
C. To perform tight alveolar packing by B. Verrucous precancer
means of iodoform tampon C. Hyperplastic form of candidosis
D. To make suture ligation of the alveolus D. Hyperkeratous form of lichen ruber
E. To introduce hemostatics planus
E. Keratoacanthoma
176. A patient with a missile wound of
his face was delivered to the station of 180. A patient had an epileptic attack
dental aid group that was organized by during tooth preparation. What actions
order of medical corps commander. What should the doctor take?
kind of aid can be rendered by the dental
aid group? A. To lay the patient down with laterad
position of his head, to fix his tongue
A. Secondary care B. No actions should be taken
B. Professional care C. To seat the patient on an armchair in
C. Consultancy upright position
D. Dental health service D. To seat the patient with forward positi-
E. Dental orthopaedic service on of his head, to fix his hindhead
E. To lay the patient in prone, head-down
177. A 13 year old child complains about position, to fix his arms
acute spontaneous short-term attack-like
pain in the area of the 36 tooth that is 181. A 62 year patient was at a denti-
getting worse during eating. The pain st’s and suddenly there appeared dyspnea
appeared yesterday. Objectively: there is and hacking cough, sense of progressi-
a deep carious cavity on the masticatory ng asphyxia. The patient’s condition is
surface of the 36 tooth. Tooth cavity is getting progressively worse; number of
closed, floor probing is painful in one poi- respiratory movements reached 30/min,
nt. Cold stimulus causes short-term pain. acrocyanosis became more evident, there
Make a diagnosis: appeared gargling breathing, oral cavi-
ty excretes a lot of albuminoid sputum.
A. Acute localized pulpitis Psychomotor agitation and arterial
B. Acute deep caries hypertension are present. What pathologi-
C. Accute diffuse pulpitis cal state is it?
D. Chronic fibrous pulpitis
E. Acute condition of chronic pulpitis A. Pulmonary edema
B. An attack of bronchial asthma
178. Mother of a 2 month old child C. Epileptic attack
had acute purulent mastitis. After that D. Anaphylactic shock
the child got edema of left infraorbital E. Spontaneous pneumothorax
and zygomatic areas, skin hyperemia of
left face part, body temperature up to 182. A patient was delivered to an admi-
39 − 40o C, purulent discharges from the ssion room after a road accident. Exami-
nose. What is presumptive diagnosis? nation of the patient revealed in the area
of oral floor a large haematoma spreading
Krok 2 Stomatology 2008 27

to the neck area as well as a significant with arthrosis of mandibulotemporal joi-


edema of soft tissues of oral floor and nt. During auscultation the doctor heard
neck. There are signs of asphyxia. What pathological rustle in the area of the joi-
type of emergency care is indicated? nt, namely "clicking"that was caused by
a load. What is the mechanism of this
A. Tracheotomy performing pathological rustle?
B. Lobeline introduction
C. Tongue fixation A. Asynchronous movement of disc and
D. Opening and draining of haematoma articulation head
E. Symptomatic medicamental treatment B. Friction of bone surfaces in the joint
C. Loosening of mandibulotemporal joint
183. A 42 year old patient applied to ligaments
an oral surgeon for the purpose of oral D. Calcification of mandibulotemporal
cavity sanitation. After anesthetization joint disc
the patient felt sudden attack of nausea, E. Inflammation of cartilaginous tissue of
weakness, he got the sense of compressi- the disc
on behind his breastbone, heart pain; he
began vomiting. The patient lost consci- 187. A 23 year old patient complained
ousness, there appeared convulsions. about unpleasant sensations in the area of
Objectively: the patient’s skin is pale, the 12 tooth, X-ray picture shows a well-
covered with cold clammy sweat, pupi- defined low-density area up to 8-10 mm in
ls don’t react to the light. The pulse diameter around the root apex of the 12
is thready, arterial pressure cannot be tooth. What method of surgery will be the
detected. What is the most probable di- most reasonable?
agnosis?
A. Root apex resection
A. Anaphylactic shock B. Root amputation
B. Collapse C. Hemisection
C. Syncope D. Tooth removal
D. Traumatic shock E. Cystotomy
E. Epileptic attack
188. A 20 year old man got a blow to the
184. A 42 year old patient complains chin. He remained conscious. Objectively:
about a neoplasm in the anterior palatal his general condition is satisfactory. He
part that has been forming for 10 years. has an edema and haematoma in the area
The 13, 12, 11, 21, 22, 23 teeth are intact. of his lower jaw on both sides. Jaw palpati-
Intraoral X-ray picture of the upper on causes acute pain. Mouth opening is
jaw shows a single focus of bony tissue limited. There is also a rupture of mucous
destruction, well-defined, 2,5 by 1,5 cm membrane of oral cavity in the area of
large. Periodontal fissure in the area of the the 44, 43 and 34, 35 teeth. Occlusion
13, 12, 11, 21, 22, 23 teeth can be well seen. is deviated. What asphyxia type can be
What is the most probable diagnosis? developed?
A. Nasopalatine canal cyst A. Dislocation
B. Radicular cyst B. Obturation
C. Globulomaxillary cyst C. Stenotic
D. Residual cyst D. Valve
E. Follicular cyst E. Aspiration
185. At the stage of adjustment of an 189. A 3 year old child was burnt with
individual tray the Herbst test should be boiling water. Face skin is hyperemic and
done. What muscles cause the shift of the edematic, there are blisters of different si-
individual tray during functional testing - zes filled with transparent liquid. Define a
alternate touching right or left cheek with degree of face skin burn:
the tip of tongue?
A. II degree
A. Mylohyoid B. I degree
B. Digastric, geniohyoid C. III degree - A
C. Mentalis and orbicular muscle of mouth D. III degree - B
D. Mimic muscles E. IV degree
E. Mastication muscles
190. Examination of an 11 year old
186. A 37 year old patient was diagnosed girl revealed: caries intensity (sum of
Krok 2 Stomatology 2008 28

carious, filled and extracted teeth) = A. Intravenous introduction of 0,5 ml of


3, Green-Vermillion’s hygienic index - 0,1% adrenalin solution, prednisolone
1,6, papillary-marginally-alveolar index = B. Promedol, suprastine
20%. The girl catches a cold once or twi- C. A cotton-wool wad soaked in liquid
ce a year. Average caries intensity index ammonia brought to the patient’s nose
for this age group in this region is 4. How D. Lying the patient in horizontal position
many times a year should the girl see a E. Aminophylline introduction
dentist?
194. A 43 year old patient applied to
A. 1 a prosthetic dentistry for the purpose
B. 1-2 of prosthetics. In anamnesis: has been
C. 2 suffering from diabetes mellitus for 15
D. 2-3 years. The patient has been very anxi-
E. 3-4 ous before the appointment with a doctor.
Suddenly the patient’s condition grew
191. A patient was admitted to the oral worse, he felt limb weakness. The patient
surgery department with a diagnosis became covered with cold clammy sweat
"odontogenous phlegmon of mouth and lost consciousness. What critical state
floor". Objectively: general condition of is it?
the patient is grave, abed position is
forced, the patient is sitting with lowered A. Hypoglycemic coma
head. He complains about pain behind his B. Syncope
breastbone that is getting worse during C. Hyperglycemic coma
backward flexion of head; cough, dyspnea. D. Anaphylactic shock
What complication has developed? E. Myocardium infarction
A. Mediastinitis 195. A patient complains about worsened
B. Phlegmon of retropharyngeal space fixation and frequent breakages of parti-
C. Cervical phlegmon al removable lamellar denture for the
D. Phlegmon of tongue root lower jaw that has been in use for
E. Phlegmon of peripharyngeal space 5 years. Objectively: alveolar part in
edentulous areas is significantly atrophi-
192. The 47, 46, 45, 35, 36, 37, 38 teeth ed, the denture balances. What is the most
of a 57 year old patient are missing. It probable cause of worsened fixation and
is planned to make a clasp denture. The frequent breakages of the denture?
48 tooth inclines to the lingual side and
forwards. On the lingual side of the 48 A. Alveolar process atrophy
tooth the border line is diagonal, on the B. Improper care of denture
buccal side it runs on a level with gingival C. Denture using during sleep
edge. What type of Neja clasp should be D. Consumption of solid food
applied? E. Wear of artificial teeth
A. V type clasp 196. A 13 year old patient got a trauma
B. I type clasp in the area of median face zone. She
C. IV type clasp (reverse back-action) complains about pain, swelling of soft
D. I-II type clasp tissues in the area of her upper jaw,
E. II type clasp pain during mouth closing. Examination
revealed mobility of nose bones, signi-
193. A patient is waiting for metal-ceramic ficant swelling of soft tissues in the left
crowns for the 11, 12 teeth. Before zygomatic area, haemorrage in the eye
preparation conduction anesthesia was sclera, "step"sign along the inferior edge
performed. A few minutes later the pati- of both orbits and zygomaticomaxillary
ent felt anxiety, ear noise, abdominal pain, sutures, nasal haemorrhage, open bite,
dizziness. Tachypnoea and rapid pulse are lengthening of median face part. Make
present. Arterial pressure is 60/40 mm Hg. a clinical diagnosis:
What kind of first aid should be rendered?
A. Le Fort’s III fracture of upper jaw
B. Le Fort’s II fracture of upper jaw
C. Le Fort’s I fracture of upper jaw
D. Fracture of nose bones
E. Fracture of skull base
197. A patient with unilateral dislocation
Krok 2 Stomatology 2008 29

of mandibulotemporal joint was delivered 199. A 25 year old patient applied to a


to the maxillofacial department. What clinic for the purpose of oral cavity sani-
type of local anesthesia is indicated for tation. During preparation of the 45 tooth
relaxation of masticatory muscles during on account of chronic median caries the
diaplasis? patient turned pale, there appeared cold
clammy sweat on her forehead, nausea,
A. Subzygomatic Bersche-Dubov- ear noise. The patient lost consciousness.
Uvarov’s anesthesia Objectively: pulse - 50 bpm, AP - 80/60
B. Intraoral mandibular anesthesia mm Hg, shallow breath, miotic pupils.
C. Extraoral mandibular anesthesia, Make a diagnosis of this state:
submandibular way
D. Extraoral mandibular anesthesia, A. Syncope
retromandibular way B. Anaphylactic shock
E. Torus anesthesia C. Stenocardia
D. Collapse
198. A patient got an injury and complai- E. Quincke’s edema
ns about limited mouth opening, nasal
haemorrhage, skin numbness in the 200. A 64 year old patient was wai-
infraorbital and inferior eyelid area. ting to see an oral surgeon. Suddenly
Objectively: there is face deformity due to she fell, there appeared coarse breathi-
retraction of soft tissues in the zygomatic ng, spasmatic twitching of upper and
area on the left, a "step"symptom in lower extremities. Face and neck became
the median part of the left inferior edge cyanotic, pupils became mydriatic, reacti-
of eyesocket as well as in the area of on to the light was absent, arteri-
zygomaticoalveolar crest. What exami- al pressure and pulse could not be
nation method should be applied for di- detected, heart tones could not be heard,
agnostics of this disease? involuntary urination was observed. What
state are these symptoms typical for?
A. Roentgenography of facial cranium in
the axial projection A. Clinical death
B. Roentgenography of facial cranium in B. Loss of consciousness
the frontal projection C. Stroke
C. Roentgenography of facial cranium in D. Myocardium infarction
the lateral projection E. Collapse
D. Roentgenography of facial cranium and
paranasal sinuses
E. Orthopantomogram

You might also like